In the AKS paper, the authors based on the Lemma
There exist constants $c > 0$ and $n_0$ such that, for all $x\ge n_0$: $$\bigl|\{q \mid \text{$q$ is prime, $q\le x$ and $P(q − 1) > q^{2/3}$}\}\bigr|\ge c\frac{x}{\ln x},,円$$ where $P(q-1)$ denotes the greatest prime divisor of number $q-1$.
to conclude that the AKS primality test will find an integer $r=O((\log n)^3)$ with $\operatorname{ord}_r(n)>(\log n)^2$.
My question is how can we determine the hidden constant in the BigOh $r=O((\log n)^3)$? I have read papers involving the AKS algorithm but none of them mention the hidden constant!
-
1$\begingroup$ Quick comment though I guess you are aware of this already. The $c$ constant you mention here comes from this paper link.springer.com/article/10.1007/BF01388980 so I guess you will have to get its value from here first, though a quick check in the paper did not allow me to conclude... $\endgroup$holf– holf2025年11月03日 14:14:30 +00:00Commented Nov 3 at 14:14
-
$\begingroup$ @NealYoung I've changed the condition. $\endgroup$minh quý lê– minh quý lê2025年11月04日 15:13:52 +00:00Commented Nov 4 at 15:13